2
$\begingroup$

This is a question taken (inferred) from Ex. 19, chap 3 in Rudin's Real and Complex Analysis book.

Let $\mu$ be Legesgue's measure on $X=[0,1]$. Given a measurable $L^{\infty}$ function $f:X\to C$, we denote by $R_f$ the essential image of $f$ (that is, the set of complex numbers $z\in C$ such that $\mu(f^{-1}(D(z,\epsilon))>0$ for all $\epsilon>0$, with $D(z,\epsilon)$ being the disc of center $z$ and radius $\epsilon$). We also denote by $A_f$ the set of all numbers of the form $z_E=\mu(E)^{-1}\int_E f\,d\mu$ for some measurable subset $E\subset [0,1]$ with $\mu(E)>0$.

It can be shown (not too difficult) that $R_f$ is a closed compact set and that we always have the inclusions $R_f\subset \bar{A_f}=\operatorname{Conv}(R_f)$, where $\operatorname{Conv}(R_f)$ denotes the convex envelope of $R_f$. (This property holds, I think, more generally for any finite, diffuse measure on a locally compact space.)

The question is: is it true that for any $f\in L^{\infty}([0,1],\mu)$, the set $A_f$ is actually convex, and if so, how to prove it?

$\endgroup$
4
  • $\begingroup$ Sorry for my mistake, $E_f$ should have read $R_f$ above (I hope this is corrected) $\endgroup$
    – jacaboul
    Jun 5, 2019 at 16:47
  • $\begingroup$ What have you tried? $\endgroup$ Jun 5, 2019 at 16:49
  • $\begingroup$ All the examples that I could think of, so far, gives a convex $A_f$. Note for example that if $f$ has a finite essential image, then the result is easy to prove. $\endgroup$
    – jacaboul
    Jun 5, 2019 at 18:47
  • $\begingroup$ The problem is that in general there might be many $z\in R_f$ that are not of the form $z_E$ for some $E$, but intuitively those points should lie on the boundary of ${\rm Conv} R_f$. More precisely I suspect one could show that $A_f$ must be locally closed (from which the result would follow), but I am so far unable to prove it. $\endgroup$
    – jacaboul
    Jun 5, 2019 at 18:54

1 Answer 1

2
$\begingroup$

Claim: For every $f \in L^1[0,1]$, the set $A_f$ is convex.

Proof: Let $\mu_1=\mu$ be Lebesgue measure on $[0,1]$ and consider the signed measures $\mu_2,\mu_3$ on $[0,1]$ defined using the real and imaginary parts of $f$ by $\mu_2(E)= \int_E {\mathrm Re} (f) \,d\mu$ and $\mu_3(E)= \int_E {\mathrm Im} (f) \,d\mu$.

We are given Lebesgue measurable sets $D,E$ in $[0,1]$ and $a,b>0$ with $a+b=1$. We must show there is a measurable set $G$ with $$(*) \quad z_G=a \cdot z_D+b \cdot z_E \, . $$ We may assume that $r:=\mu(D)/\mu(E) \le 1$. By the Lyapunov theorem on convexity of the range of nonatomic vector measures (see references below; it applies to signed measures, see e.g. [4]) there is a measurable set $E_1$ in $[0,1]$ with $$(\mu_1,\mu_2,\mu_3)(E_1)=r \cdot (\mu_1,\mu_2,\mu_3)(E) +(1-r) \cdot(\mu_1,\mu_2,\mu_3)(\emptyset) \, , $$ so $\mu(E_1)=\mu(D)$ and $z_{E_1}=z_E$. By another application of Lyapunov's theorem, there is a measurable set $G$ in $[0,1]$ with $$ (\mu_1,\mu_2,\mu_3)(G)=a \cdot(\mu_1,\mu_2,\mu_3)(D)+b \cdot (\mu_1,\mu_2,\mu_3)(E_1) \,. $$ Clearly $\mu(G)=\mu(D)=\mu(E_1)$ and $G$ satisfies (*).

References:

[1] A. Liapounoff, Sur les fonctions-vecteurs compl6tement additives, Bull. Acad. Sci. URSS S6r. Math. [Izvestia Akad. Nauk SSSR] 4 (1940) 465-478.

[2] J. Lindenstrauss, A short proof of Liapounoff's convexity theorem, J. Math. Mech. 15 (1966) 971-972

[3] https://en.wikipedia.org/wiki/Vector_measure#Lyapunov

[4] Artstein, Zvi. "Yet another proof of the Lyapunov convexity theorem." Proceedings of the American Mathematical Society 108, no. 1 (1990): 89-91. https://www.ams.org/journals/proc/1990-108-01/S0002-9939-1990-0993737-0/S0002-9939-1990-0993737-0.pdf

[5] Ross, David A. "An elementary proof of Lyapunov's theorem." The American Mathematical Monthly 112, no. 7 (2005): 651-653.

$\endgroup$
3
  • $\begingroup$ Thanks for noticing the typo. No normalization needed. Corrected now. $\endgroup$ Jun 6, 2019 at 16:02
  • $\begingroup$ Many thanks for this elegant proof, and for pointing out the link to Lyapunov's theorem. There is one misprint above: the factors $\mu(E)^{-1}$ in the definition of $\mu_2$ and $\mu_3$ are to be removed. $\endgroup$
    – jacaboul
    Jun 6, 2019 at 16:40
  • $\begingroup$ Thanks Jacaboul, I already removed these factors following the comment from @Nate Eldredge - can you accept the answer now? $\endgroup$ Jun 6, 2019 at 18:37

Your Answer

By clicking “Post Your Answer”, you agree to our terms of service and acknowledge you have read our privacy policy.

Not the answer you're looking for? Browse other questions tagged or ask your own question.